prove this sequence is increasing

The name of the pictureThe name of the pictureThe name of the pictureClash Royale CLAN TAG#URR8PPP











up vote
2
down vote

favorite












The sequence $A_n$ is described as-
beginalign*
A_1 & = sqrt2\
A_n+1 & =(2+A_n)^0.5
endalign*

where $A_n$ is the $n$th term in the sequence.



I can prove by induction that this sequence is bounded by above by $2$ and then find the limit to be $2$. But in order to assume that the limit exists, I need to show that it is also increasing so I can use the monotone convergence theorem. It is easy to see why it is increasing, but how do I prove it?










share|cite|improve this question























  • Please read this tutorial on how to typeset mathematics on this site.
    – N. F. Taussig
    1 hour ago














up vote
2
down vote

favorite












The sequence $A_n$ is described as-
beginalign*
A_1 & = sqrt2\
A_n+1 & =(2+A_n)^0.5
endalign*

where $A_n$ is the $n$th term in the sequence.



I can prove by induction that this sequence is bounded by above by $2$ and then find the limit to be $2$. But in order to assume that the limit exists, I need to show that it is also increasing so I can use the monotone convergence theorem. It is easy to see why it is increasing, but how do I prove it?










share|cite|improve this question























  • Please read this tutorial on how to typeset mathematics on this site.
    – N. F. Taussig
    1 hour ago












up vote
2
down vote

favorite









up vote
2
down vote

favorite











The sequence $A_n$ is described as-
beginalign*
A_1 & = sqrt2\
A_n+1 & =(2+A_n)^0.5
endalign*

where $A_n$ is the $n$th term in the sequence.



I can prove by induction that this sequence is bounded by above by $2$ and then find the limit to be $2$. But in order to assume that the limit exists, I need to show that it is also increasing so I can use the monotone convergence theorem. It is easy to see why it is increasing, but how do I prove it?










share|cite|improve this question















The sequence $A_n$ is described as-
beginalign*
A_1 & = sqrt2\
A_n+1 & =(2+A_n)^0.5
endalign*

where $A_n$ is the $n$th term in the sequence.



I can prove by induction that this sequence is bounded by above by $2$ and then find the limit to be $2$. But in order to assume that the limit exists, I need to show that it is also increasing so I can use the monotone convergence theorem. It is easy to see why it is increasing, but how do I prove it?







real-analysis sequences-and-series






share|cite|improve this question















share|cite|improve this question













share|cite|improve this question




share|cite|improve this question








edited 1 hour ago









N. F. Taussig

42.1k93254




42.1k93254










asked 2 hours ago









childishsadbino

395




395











  • Please read this tutorial on how to typeset mathematics on this site.
    – N. F. Taussig
    1 hour ago
















  • Please read this tutorial on how to typeset mathematics on this site.
    – N. F. Taussig
    1 hour ago















Please read this tutorial on how to typeset mathematics on this site.
– N. F. Taussig
1 hour ago




Please read this tutorial on how to typeset mathematics on this site.
– N. F. Taussig
1 hour ago










4 Answers
4






active

oldest

votes

















up vote
2
down vote













Prove by induction that $A_n leq 2$. Then note that $(A_n-frac 1 2)^2 leq frac 9 4$. When expanded this reduces to $A_n+1 geq A_n$.






share|cite|improve this answer



























    up vote
    2
    down vote













    We need to prove that



    $$A_n+1=sqrt2+A_nge A_n iff A_n^2-A_n-2 le 0 iff -1le A_n le2$$



    then we have $A_n ge 0$ and by inducion we can prove that $A_nle 2$ indeed



    • base case: $A_1=sqrt 2le 2$

    • induction step: assuming true $A_nle 2$ we need to prove that $A_n+1le 2$ then

    $$A_n+1=sqrt2+A_n lesqrt2+2=2$$






    share|cite|improve this answer



























      up vote
      1
      down vote













      If we can assume $A_n, A_n+1 > 0$ then



      $A_n+1 > A_n iff A_n+1^2 > A_n^2$



      And $A_n+1^2n = ((2 + A_n)^frac 12) = 2 + A_n$



      If we can assume $A_n < 2$ then $A_n+1^2 = 2 + A_n > A_n + A_n > 2*A_n > A_n*A_n = A_n^2$.



      And that's that.



      .....



      We do have to prove that $A_n > 0$ but that's trivial:



      By induction: Base case: $sqrt 2 > 0$. Induction step. If $A_n >0$ then $A_n + 2 > 0$ so $A_n+1 > sqrtA_n + 2 > 0$.



      And we have to prove $A_n < 2$ but that's easy:



      By induction: Base case: $sqrt 2 <2$. Induction step. If $A_n < 2$ then $A_n + 2 < 4$ and $A_n+1 = sqrt2+A_n < sqrt 4 = 2$.



      =====



      If we start from the begining we can to it is one fell swoop:



      Proposition: $0 < A_1 < A_2 < .... A_n < A_n+1 <.... < 2$.



      Base Case: $0 < A_1 = sqrt 2 < 2$



      Induction case: Suppose $0 < A_n < 2$ then



      $0 < A_n^2 < 2A_n=A_n + A_n < A_n + 2 < 2+2=4$



      so $0< sqrtA_n^2 < sqrtA_n + 2 < sqrt4$



      and $0< A_n < A_n+1 < 2$.






      share|cite|improve this answer



























        up vote
        1
        down vote













        A proof by contradiction.



        Suppose to the contrary that for some term, we have $A_k+1 leq A_k$. Suppose that this is the first instance for which this occurs. Now, we have that



        $$A_k+1 = sqrt2+A_k,$$ so that by using our assumption,
        $$ 2 + A_k= A_k+1^2 leq A_k^2.$$



        Now, since $$A_k = sqrt2+A_k-1,$$
        then also $$A_k^2 = 2+A_k-1.$$
        So, plugging in above, we have that
        $$2+A_k leq A_k^2 = 2+A_k-1.$$



        In other words, $$A_k leq A_k-1.$$



        This is a contradiction to the assumption that $A_k+1$ was the first instance for which this occurs. Thus, the sequence must be increasing.






        share|cite|improve this answer




















          Your Answer





          StackExchange.ifUsing("editor", function ()
          return StackExchange.using("mathjaxEditing", function ()
          StackExchange.MarkdownEditor.creationCallbacks.add(function (editor, postfix)
          StackExchange.mathjaxEditing.prepareWmdForMathJax(editor, postfix, [["$", "$"], ["\\(","\\)"]]);
          );
          );
          , "mathjax-editing");

          StackExchange.ready(function()
          var channelOptions =
          tags: "".split(" "),
          id: "69"
          ;
          initTagRenderer("".split(" "), "".split(" "), channelOptions);

          StackExchange.using("externalEditor", function()
          // Have to fire editor after snippets, if snippets enabled
          if (StackExchange.settings.snippets.snippetsEnabled)
          StackExchange.using("snippets", function()
          createEditor();
          );

          else
          createEditor();

          );

          function createEditor()
          StackExchange.prepareEditor(
          heartbeatType: 'answer',
          convertImagesToLinks: true,
          noModals: true,
          showLowRepImageUploadWarning: true,
          reputationToPostImages: 10,
          bindNavPrevention: true,
          postfix: "",
          imageUploader:
          brandingHtml: "Powered by u003ca class="icon-imgur-white" href="https://imgur.com/"u003eu003c/au003e",
          contentPolicyHtml: "User contributions licensed under u003ca href="https://creativecommons.org/licenses/by-sa/3.0/"u003ecc by-sa 3.0 with attribution requiredu003c/au003e u003ca href="https://stackoverflow.com/legal/content-policy"u003e(content policy)u003c/au003e",
          allowUrls: true
          ,
          noCode: true, onDemand: true,
          discardSelector: ".discard-answer"
          ,immediatelyShowMarkdownHelp:true
          );



          );













           

          draft saved


          draft discarded


















          StackExchange.ready(
          function ()
          StackExchange.openid.initPostLogin('.new-post-login', 'https%3a%2f%2fmath.stackexchange.com%2fquestions%2f2990748%2fprove-this-sequence-is-increasing%23new-answer', 'question_page');

          );

          Post as a guest






























          4 Answers
          4






          active

          oldest

          votes








          4 Answers
          4






          active

          oldest

          votes









          active

          oldest

          votes






          active

          oldest

          votes








          up vote
          2
          down vote













          Prove by induction that $A_n leq 2$. Then note that $(A_n-frac 1 2)^2 leq frac 9 4$. When expanded this reduces to $A_n+1 geq A_n$.






          share|cite|improve this answer
























            up vote
            2
            down vote













            Prove by induction that $A_n leq 2$. Then note that $(A_n-frac 1 2)^2 leq frac 9 4$. When expanded this reduces to $A_n+1 geq A_n$.






            share|cite|improve this answer






















              up vote
              2
              down vote










              up vote
              2
              down vote









              Prove by induction that $A_n leq 2$. Then note that $(A_n-frac 1 2)^2 leq frac 9 4$. When expanded this reduces to $A_n+1 geq A_n$.






              share|cite|improve this answer












              Prove by induction that $A_n leq 2$. Then note that $(A_n-frac 1 2)^2 leq frac 9 4$. When expanded this reduces to $A_n+1 geq A_n$.







              share|cite|improve this answer












              share|cite|improve this answer



              share|cite|improve this answer










              answered 2 hours ago









              Kavi Rama Murthy

              37k31746




              37k31746




















                  up vote
                  2
                  down vote













                  We need to prove that



                  $$A_n+1=sqrt2+A_nge A_n iff A_n^2-A_n-2 le 0 iff -1le A_n le2$$



                  then we have $A_n ge 0$ and by inducion we can prove that $A_nle 2$ indeed



                  • base case: $A_1=sqrt 2le 2$

                  • induction step: assuming true $A_nle 2$ we need to prove that $A_n+1le 2$ then

                  $$A_n+1=sqrt2+A_n lesqrt2+2=2$$






                  share|cite|improve this answer
























                    up vote
                    2
                    down vote













                    We need to prove that



                    $$A_n+1=sqrt2+A_nge A_n iff A_n^2-A_n-2 le 0 iff -1le A_n le2$$



                    then we have $A_n ge 0$ and by inducion we can prove that $A_nle 2$ indeed



                    • base case: $A_1=sqrt 2le 2$

                    • induction step: assuming true $A_nle 2$ we need to prove that $A_n+1le 2$ then

                    $$A_n+1=sqrt2+A_n lesqrt2+2=2$$






                    share|cite|improve this answer






















                      up vote
                      2
                      down vote










                      up vote
                      2
                      down vote









                      We need to prove that



                      $$A_n+1=sqrt2+A_nge A_n iff A_n^2-A_n-2 le 0 iff -1le A_n le2$$



                      then we have $A_n ge 0$ and by inducion we can prove that $A_nle 2$ indeed



                      • base case: $A_1=sqrt 2le 2$

                      • induction step: assuming true $A_nle 2$ we need to prove that $A_n+1le 2$ then

                      $$A_n+1=sqrt2+A_n lesqrt2+2=2$$






                      share|cite|improve this answer












                      We need to prove that



                      $$A_n+1=sqrt2+A_nge A_n iff A_n^2-A_n-2 le 0 iff -1le A_n le2$$



                      then we have $A_n ge 0$ and by inducion we can prove that $A_nle 2$ indeed



                      • base case: $A_1=sqrt 2le 2$

                      • induction step: assuming true $A_nle 2$ we need to prove that $A_n+1le 2$ then

                      $$A_n+1=sqrt2+A_n lesqrt2+2=2$$







                      share|cite|improve this answer












                      share|cite|improve this answer



                      share|cite|improve this answer










                      answered 1 hour ago









                      gimusi

                      82.6k74091




                      82.6k74091




















                          up vote
                          1
                          down vote













                          If we can assume $A_n, A_n+1 > 0$ then



                          $A_n+1 > A_n iff A_n+1^2 > A_n^2$



                          And $A_n+1^2n = ((2 + A_n)^frac 12) = 2 + A_n$



                          If we can assume $A_n < 2$ then $A_n+1^2 = 2 + A_n > A_n + A_n > 2*A_n > A_n*A_n = A_n^2$.



                          And that's that.



                          .....



                          We do have to prove that $A_n > 0$ but that's trivial:



                          By induction: Base case: $sqrt 2 > 0$. Induction step. If $A_n >0$ then $A_n + 2 > 0$ so $A_n+1 > sqrtA_n + 2 > 0$.



                          And we have to prove $A_n < 2$ but that's easy:



                          By induction: Base case: $sqrt 2 <2$. Induction step. If $A_n < 2$ then $A_n + 2 < 4$ and $A_n+1 = sqrt2+A_n < sqrt 4 = 2$.



                          =====



                          If we start from the begining we can to it is one fell swoop:



                          Proposition: $0 < A_1 < A_2 < .... A_n < A_n+1 <.... < 2$.



                          Base Case: $0 < A_1 = sqrt 2 < 2$



                          Induction case: Suppose $0 < A_n < 2$ then



                          $0 < A_n^2 < 2A_n=A_n + A_n < A_n + 2 < 2+2=4$



                          so $0< sqrtA_n^2 < sqrtA_n + 2 < sqrt4$



                          and $0< A_n < A_n+1 < 2$.






                          share|cite|improve this answer
























                            up vote
                            1
                            down vote













                            If we can assume $A_n, A_n+1 > 0$ then



                            $A_n+1 > A_n iff A_n+1^2 > A_n^2$



                            And $A_n+1^2n = ((2 + A_n)^frac 12) = 2 + A_n$



                            If we can assume $A_n < 2$ then $A_n+1^2 = 2 + A_n > A_n + A_n > 2*A_n > A_n*A_n = A_n^2$.



                            And that's that.



                            .....



                            We do have to prove that $A_n > 0$ but that's trivial:



                            By induction: Base case: $sqrt 2 > 0$. Induction step. If $A_n >0$ then $A_n + 2 > 0$ so $A_n+1 > sqrtA_n + 2 > 0$.



                            And we have to prove $A_n < 2$ but that's easy:



                            By induction: Base case: $sqrt 2 <2$. Induction step. If $A_n < 2$ then $A_n + 2 < 4$ and $A_n+1 = sqrt2+A_n < sqrt 4 = 2$.



                            =====



                            If we start from the begining we can to it is one fell swoop:



                            Proposition: $0 < A_1 < A_2 < .... A_n < A_n+1 <.... < 2$.



                            Base Case: $0 < A_1 = sqrt 2 < 2$



                            Induction case: Suppose $0 < A_n < 2$ then



                            $0 < A_n^2 < 2A_n=A_n + A_n < A_n + 2 < 2+2=4$



                            so $0< sqrtA_n^2 < sqrtA_n + 2 < sqrt4$



                            and $0< A_n < A_n+1 < 2$.






                            share|cite|improve this answer






















                              up vote
                              1
                              down vote










                              up vote
                              1
                              down vote









                              If we can assume $A_n, A_n+1 > 0$ then



                              $A_n+1 > A_n iff A_n+1^2 > A_n^2$



                              And $A_n+1^2n = ((2 + A_n)^frac 12) = 2 + A_n$



                              If we can assume $A_n < 2$ then $A_n+1^2 = 2 + A_n > A_n + A_n > 2*A_n > A_n*A_n = A_n^2$.



                              And that's that.



                              .....



                              We do have to prove that $A_n > 0$ but that's trivial:



                              By induction: Base case: $sqrt 2 > 0$. Induction step. If $A_n >0$ then $A_n + 2 > 0$ so $A_n+1 > sqrtA_n + 2 > 0$.



                              And we have to prove $A_n < 2$ but that's easy:



                              By induction: Base case: $sqrt 2 <2$. Induction step. If $A_n < 2$ then $A_n + 2 < 4$ and $A_n+1 = sqrt2+A_n < sqrt 4 = 2$.



                              =====



                              If we start from the begining we can to it is one fell swoop:



                              Proposition: $0 < A_1 < A_2 < .... A_n < A_n+1 <.... < 2$.



                              Base Case: $0 < A_1 = sqrt 2 < 2$



                              Induction case: Suppose $0 < A_n < 2$ then



                              $0 < A_n^2 < 2A_n=A_n + A_n < A_n + 2 < 2+2=4$



                              so $0< sqrtA_n^2 < sqrtA_n + 2 < sqrt4$



                              and $0< A_n < A_n+1 < 2$.






                              share|cite|improve this answer












                              If we can assume $A_n, A_n+1 > 0$ then



                              $A_n+1 > A_n iff A_n+1^2 > A_n^2$



                              And $A_n+1^2n = ((2 + A_n)^frac 12) = 2 + A_n$



                              If we can assume $A_n < 2$ then $A_n+1^2 = 2 + A_n > A_n + A_n > 2*A_n > A_n*A_n = A_n^2$.



                              And that's that.



                              .....



                              We do have to prove that $A_n > 0$ but that's trivial:



                              By induction: Base case: $sqrt 2 > 0$. Induction step. If $A_n >0$ then $A_n + 2 > 0$ so $A_n+1 > sqrtA_n + 2 > 0$.



                              And we have to prove $A_n < 2$ but that's easy:



                              By induction: Base case: $sqrt 2 <2$. Induction step. If $A_n < 2$ then $A_n + 2 < 4$ and $A_n+1 = sqrt2+A_n < sqrt 4 = 2$.



                              =====



                              If we start from the begining we can to it is one fell swoop:



                              Proposition: $0 < A_1 < A_2 < .... A_n < A_n+1 <.... < 2$.



                              Base Case: $0 < A_1 = sqrt 2 < 2$



                              Induction case: Suppose $0 < A_n < 2$ then



                              $0 < A_n^2 < 2A_n=A_n + A_n < A_n + 2 < 2+2=4$



                              so $0< sqrtA_n^2 < sqrtA_n + 2 < sqrt4$



                              and $0< A_n < A_n+1 < 2$.







                              share|cite|improve this answer












                              share|cite|improve this answer



                              share|cite|improve this answer










                              answered 1 hour ago









                              fleablood

                              64.8k22680




                              64.8k22680




















                                  up vote
                                  1
                                  down vote













                                  A proof by contradiction.



                                  Suppose to the contrary that for some term, we have $A_k+1 leq A_k$. Suppose that this is the first instance for which this occurs. Now, we have that



                                  $$A_k+1 = sqrt2+A_k,$$ so that by using our assumption,
                                  $$ 2 + A_k= A_k+1^2 leq A_k^2.$$



                                  Now, since $$A_k = sqrt2+A_k-1,$$
                                  then also $$A_k^2 = 2+A_k-1.$$
                                  So, plugging in above, we have that
                                  $$2+A_k leq A_k^2 = 2+A_k-1.$$



                                  In other words, $$A_k leq A_k-1.$$



                                  This is a contradiction to the assumption that $A_k+1$ was the first instance for which this occurs. Thus, the sequence must be increasing.






                                  share|cite|improve this answer
























                                    up vote
                                    1
                                    down vote













                                    A proof by contradiction.



                                    Suppose to the contrary that for some term, we have $A_k+1 leq A_k$. Suppose that this is the first instance for which this occurs. Now, we have that



                                    $$A_k+1 = sqrt2+A_k,$$ so that by using our assumption,
                                    $$ 2 + A_k= A_k+1^2 leq A_k^2.$$



                                    Now, since $$A_k = sqrt2+A_k-1,$$
                                    then also $$A_k^2 = 2+A_k-1.$$
                                    So, plugging in above, we have that
                                    $$2+A_k leq A_k^2 = 2+A_k-1.$$



                                    In other words, $$A_k leq A_k-1.$$



                                    This is a contradiction to the assumption that $A_k+1$ was the first instance for which this occurs. Thus, the sequence must be increasing.






                                    share|cite|improve this answer






















                                      up vote
                                      1
                                      down vote










                                      up vote
                                      1
                                      down vote









                                      A proof by contradiction.



                                      Suppose to the contrary that for some term, we have $A_k+1 leq A_k$. Suppose that this is the first instance for which this occurs. Now, we have that



                                      $$A_k+1 = sqrt2+A_k,$$ so that by using our assumption,
                                      $$ 2 + A_k= A_k+1^2 leq A_k^2.$$



                                      Now, since $$A_k = sqrt2+A_k-1,$$
                                      then also $$A_k^2 = 2+A_k-1.$$
                                      So, plugging in above, we have that
                                      $$2+A_k leq A_k^2 = 2+A_k-1.$$



                                      In other words, $$A_k leq A_k-1.$$



                                      This is a contradiction to the assumption that $A_k+1$ was the first instance for which this occurs. Thus, the sequence must be increasing.






                                      share|cite|improve this answer












                                      A proof by contradiction.



                                      Suppose to the contrary that for some term, we have $A_k+1 leq A_k$. Suppose that this is the first instance for which this occurs. Now, we have that



                                      $$A_k+1 = sqrt2+A_k,$$ so that by using our assumption,
                                      $$ 2 + A_k= A_k+1^2 leq A_k^2.$$



                                      Now, since $$A_k = sqrt2+A_k-1,$$
                                      then also $$A_k^2 = 2+A_k-1.$$
                                      So, plugging in above, we have that
                                      $$2+A_k leq A_k^2 = 2+A_k-1.$$



                                      In other words, $$A_k leq A_k-1.$$



                                      This is a contradiction to the assumption that $A_k+1$ was the first instance for which this occurs. Thus, the sequence must be increasing.







                                      share|cite|improve this answer












                                      share|cite|improve this answer



                                      share|cite|improve this answer










                                      answered 1 hour ago









                                      Euler....IS_ALIVE

                                      2,59311336




                                      2,59311336



























                                           

                                          draft saved


                                          draft discarded















































                                           


                                          draft saved


                                          draft discarded














                                          StackExchange.ready(
                                          function ()
                                          StackExchange.openid.initPostLogin('.new-post-login', 'https%3a%2f%2fmath.stackexchange.com%2fquestions%2f2990748%2fprove-this-sequence-is-increasing%23new-answer', 'question_page');

                                          );

                                          Post as a guest













































































                                          Comments

                                          Popular posts from this blog

                                          Long meetings (6-7 hours a day): Being “babysat” by supervisor

                                          Is the Concept of Multiple Fantasy Races Scientifically Flawed? [closed]

                                          Confectionery